LSAT and Law School Admissions Forum

Get expert LSAT preparation and law school admissions advice from PowerScore Test Preparation.

User avatar
 Dave Killoran
PowerScore Staff
  • PowerScore Staff
  • Posts: 5852
  • Joined: Mar 25, 2011
|
#27115
Complete Question Explanation
(The complete setup for this game can be found here: lsat/viewtopic.php?t=49)

The correct answer choice is (B)

As we determined from our discussion of the inferences, when P is carried then T must be carried. This relationship eliminates answer choices (A), (C), and (E).

Also, when O is carried then P and W must be carried, which eliminates answer choices (A) (again) and (D).

Of course, K and P cannot be carried together, and this eliminates (E) as well.

Consequently, answer choice (B) is proven correct by process of elimination.
User avatar
 mariela45
  • Posts: 2
  • Joined: Aug 08, 2021
|
#89792
I STILL do not understand
User avatar
 atierney
PowerScore Staff
  • PowerScore Staff
  • Posts: 215
  • Joined: Jul 06, 2021
|
#89831
Hi Mariela,

This is the first question (of the entire LG section no less), and it is the global could be true question as to a possible arrangement (solution) to the game. There are a number of ways to tackle this question, and most students seem to prefer going through each rule and eliminating possible answer choices where they find a conflict or violation of a particular rule. I definitely think this is foolproof means of getting the answer, but you also might try to analyze each particular answer against the entirety of the rules at once. In other words, take an individual answer choice (rather than an individual rule) and compare off the checklist you have compiled of the rules themselves. This was the method that I employed when I took the LSAT, and the advantage (which is slight) that it has is that you are just looking for the first answer that doesn't violate any rules, i.e. passes through all the checkpoints. Of course, alternatively, any inferences that you've made already can be used to quickly eliminate flagrant rule violations, thus you may use hybrid of these two methods. At the end of the day, I think it's a good idea to establish a particular strategy that you trust will work and employ that for each of these questions. They really some of the more straightforward on the LG section as a whole.

Now, having said that, for this particular question, from a game featured in our course book, if you employed my method, your first step would be to examine answer choice A, which is not correct given the second rule, which states that kiwis must be carried when tangerines are not, thus we have to either see tangerines in the answer choice or kiwis (or both). Moving on to answer choice B, tangerines appears, thus negating the need to carry kiwis, and additionally pears also appear. This is a valid solution that does not run afoul of any rules and thus is the correct answer for this Could Be True question. We can check the other rules, particularly for any complications with pears in the answer choice; however, once convinced that B doesn't run afoul of any rules, it will necessarily be the only one so designated, and thus, we can be confident that it is the correct answer.

Let me know if you have further questions on this, either the method for choosing B or why B is correct.

Get the most out of your LSAT Prep Plus subscription.

Analyze and track your performance with our Testing and Analytics Package.